http://7sage.com/lsat_explanations/lsat-17-section-3-question-15/
Kinda confused about this one, and why the anser is B. What does the vocal tract have to do with anything?
... 7sage.com/lsat_explanations/lsat-33-section-3-question-08/
In JY's ... lecture, for the group 3 logical indicators such as " ... sufficient condition. So for this question, I first negate "doing research ... I got wrong on this question.
http://7sage.com/lsat_explanations/lsat-17-section-3-question-19/
Why is A incorrect? Is it because having the tallest peaks in the region doesn't mean it has the absolute tallest peaks?
... ://7sage.com/lsat_explanations/lsat-33-section-3-question-08/
I think I ... may have answered this question myself, but can you guys ... , since this is an inference question I should be taking all ...
... .com/lsat_explanations/lsat-25-section-2-question-19/
This is ... a most strongly supported except question. This was a really ... time trying to redo this question. This is supported. When ... this difficult? I know PT28-Section3-Question 4 (from the MSS ...
... ://7sage.com/lsat_explanations/lsat-40-section-3-question-18/
Between Answer Choice ... />
I am looking at this question thinking ok, Historians do not ...
http://7sage.com/lsat_explanations/lsat-22-section-2-question-16/
Hello, I was wondering if someone could help me understand why D is incorrect? Thanks! :)
http://7sage.com/lsat_explanations/lsat-51-section-3-question-18/
Ok. The explanation JY wrote is awesome, but I still don't understand why B is correct.
http://7sage.com/lsat_explanations/lsat-59-section-3-question-24/
Answer is E. What does this even mean? :O
E) "presumes,without providing justification, that 18 century European aesthetics is as encompassing as an aesthetic theory can be"
... ://7sage.com/lsat_explanations/lsat-46-section-3-question-24/
This sufficient assumption ... question really has me thrown. I' ... time with understanding the whole question.
Hey 7Sagers! Here's a question from a student I thought ... attempted negations for necessary assumption question Dec 2009, s3,q25. Thank ... ://7sage.com/lsat_explanations/lsat-59-section-3-question-25/
Clearly I'm having a hard time mapping out logic... Can someone please explain the correct answer? Also any tips on how to improve mapping out logic would be greatly appreciated!! Thanks!
Tough. Choice (E) is correct.
You can tell by using the
contrapositive on both of these statements:
If not (prices fall as rapidly as/more rapidly than
competitors),
then not (production costs fall as rapidly or more ...
... ://7sage.com/lsat_explanations/lsat-56-section-3-question-21
I understand Jon ... 's explanation on this question, but I was tricked by ... the question's word choice "revival" in ...